Question

Q1. Two rigid members (BCDE and EKN) are connected by a pin at E. The structure is supported by a pin at B and two rollers at
0 0
Add a comment Improve this question Transcribed image text
Answer #1

6.67 kr 96 9 64 24 64 ( These rear-ho 07 e the three c i Celcwleated K. 96 2M 2. (4フ SFO 96賊 64[u4 Mu,o(ve value) 129 kn let g C has lengv a, f๓ egual aLs 6h.de. veve value 962 128 1.33

Add a comment
Know the answer?
Add Answer to:
Q1. Two rigid members (BCDE and EKN) are connected by a pin at E. The structure is supported by a...
Your Answer:

Post as a guest

Your Name:

What's your source?

Earn Coins

Coins can be redeemed for fabulous gifts.

Not the answer you're looking for? Ask your own homework help question. Our experts will answer your question WITHIN MINUTES for Free.
Similar Homework Help Questions
  • 5) Two rigid members (AE and ABCD) are connected by a pin at A. CK is another rigid bar supported...

    5) Two rigid members (AE and ABCD) are connected by a pin at A. CK is another rigid bar supported by a fixed-end at K. The contact at C is frictionless and D is and the tension on the cable AH is 52 kN, a roller. If the system is in equilibrium a) draw free body diagrams of all rigid members (AE, ABCD and CK), b) determine P (kN) w (kN/m), all reaction forces (or force components) at A, C,...

  • The structure below is supported by a pin at A and a roller at E Determine the force in each memb...

    The structure below is supported by a pin at A and a roller at E Determine the force in each member of the truss using the Method of Joints H meters 2 FE m AH m GF m HG m Joint Loadings, kN ent length, m Height, m HG GF FE 1.5 2 Draw FBD of truss system to determine reaction forces Assume all forces in members are tensile. If answer is negative, force is compressive. s in x direction...

  • The beam AC is supported by a smooth pin at A and a roller at B...

    The beam AC is supported by a smooth pin at A and a roller at B as shown in the figure below. a. Sketch the free-body diagram of the beam and use it to determine the support reaction components at A and B. b. Draw the shear and moment diagrams for the beam. 6. The beam AC is supported by a smooth pin at A and a roller at B as shown in the figure below. 6 kN 12 kN/m...

  • W The figure shows a frame consisting of two members joined together at pin B and...

    W The figure shows a frame consisting of two members joined together at pin B and supported at C and D. a) Draw the free body diagram of member ABD. b) Calculate the force applied by pin B on the member. c) Calculate the forces at D. OD (hint: member BC is an axial member) Aa 4m 8m 6 kN

  • QUESTION 2 Beam ABCD is 8 m in length and is pin-supported at A and roller-supported...

    QUESTION 2 Beam ABCD is 8 m in length and is pin-supported at A and roller-supported at C as shown in Figure Q2. A counter-clockwise concentrated moment acts about the support A. A uniformly-distributed load acts on span BC and a vertical concentrated load acts at the free end D a) Determine the reactions at supports A and C. 4 marks) b) Obtain the shear force and the bending moment functions (in terms of x) for each segment along the...

  • 1) The uniform beam shown is supported by a pin at A and a light rope...

    1) The uniform beam shown is supported by a pin at A and a light rope at B. A 1,000 lb weight is supported at C. Determine the normal force, shear force, and bending moment at point P. (15 p.) 30 3 А 2) The uniform beam shown is supported by a pin at and a roller at B. Using the analytical method (i.e., sections), construct the shear and moment diagrams. Write your equations V(x) and Mix) for each section...

  • My answers are incorrect. need help The structure shown below is supported by a pin and...

    My answers are incorrect. need help The structure shown below is supported by a pin and roller on an inclined surface that is parallel to the line connecting joints A and G. Given that P= 480 lb. 18 in.+ 18 in.+18 in. +18 in. PI Тc Determine the force supported by the members CD, CI, and IJ. The force supported by the members CD, CI, and IJ is as follows: FCD=-349.602 lb Fci=-224.322 1b Fiy=[ 901.904 1b

  • The rigid frame shown below is supported by Pin A and Roller C.

    The rigid frame shown below is supported by Pin A and Roller C. [Point B is a rigid joint.] The frame supports a uniformly distributed load of 20 kN/m (downward) in Region BC, and a 250 kN point load (downward) located halfway between Pin A and rigid joint B. The modulus of elasticity of the entire frame is E = 200 GPa and the moment of inertia is I = 500 x 106 mm4. Determine the rotation (slope) at Joint...

  • A is a roller and C is pin connected 3-9. Determine the force in each member...

    A is a roller and C is pin connected 3-9. Determine the force in each member of the truss. State if the members are in tension or compression. E 12 kN 8 kN 3 m 45° 24 kN 4 m Prob. 3-9

  • Draw the Shear Force (V) and Bending Moment (MI) diagrams of statically indeterminate beam shown in...

    Draw the Shear Force (V) and Bending Moment (MI) diagrams of statically indeterminate beam shown in figure using “Force Method”. The (roller) support at "B" settles 35 mm. The moment of inertia is given by (1) for regions "AB", "BC" and "CD"; however it is equal to (21) for the region “DE”. ("B" is the roller and “E" is the fixed type of support). [The flexural rigidity: EI=40000 kNm] 60 KN y 10 kN/m A - Tu (21) 1.5m 11...

ADVERTISEMENT
Free Homework Help App
Download From Google Play
Scan Your Homework
to Get Instant Free Answers
Need Online Homework Help?
Ask a Question
Get Answers For Free
Most questions answered within 3 hours.
ADVERTISEMENT
ADVERTISEMENT
ADVERTISEMENT